I got it right, but why?
I got this one correct through the process of elimination, but I am not 100% certain why it is co...
allbirdsarescooters Tuesday at 12:41AM
  • September 2009 LSAT
  • SEC5
  • Q17
2
Replies
Why is answer choice D incorrect here?
The stimulus states that ALL pythons that contract the disease die within 6 months. The contrapos...
Martin-DiMarco on April 15, 2022
  • September 2009 LSAT
  • SEC5
  • Q9
1
Reply
Why choice B is a correct answer itself
I can eliminate other choices however, as to choice B, isn't volcanic event is one of the natural...
Frank on June 17, 2021
  • September 2009 LSAT
  • SEC5
  • Q23
8
Replies
(B)
Hi LSAT Max, I chose (B) and was wondering why it would be incorrect. Can we eliminate this c...
Julie-V on April 30, 2021
  • September 2009 LSAT
  • SEC5
  • Q20
3
Replies
September 2009 SEC 5 Q20
I'm still quite confused with this question. The use of language such as abnormally low in the an...
kens on April 30, 2021
  • September 2009 LSAT
  • SEC5
  • Q20
3
Replies
How is volcanic activity not a natural cause?
How is volcanic activity not a natural cause?
Maria-Marin on February 3, 2021
  • September 2009 LSAT
  • SEC5
  • Q23
2
Replies
September 2009 SEC 5 Q13
Can you explain why B is incorrect? How can we know the climate condition in other regions within...
kens on June 28, 2020
  • September 2009 LSAT
  • SEC5
  • Q13
1
Reply
September 2009 SEC 5 Q9
How can we be sure that liver disease in North-American pythons will not be detected and treated?...
kens on June 28, 2020
  • September 2009 LSAT
  • SEC5
  • Q9
1
Reply
Why isn’t A correct?
I got the answer correct, but I’m not sure why A wouldn’t also be correct. Could someone explain?
bcross on June 26, 2020
  • September 2009 LSAT
  • SEC5
  • Q14
1
Reply
Why wouldn’t E be the correct answer?
I was stuck between A and E and chose wrong. Could you explain why A is correct? Also, I thought...
bcross on June 26, 2020
  • September 2009 LSAT
  • SEC5
  • Q12
3
Replies
Answer choice A
Why is A wrong and how does B strengthen the argument?
Mnv on June 19, 2020
  • September 2009 LSAT
  • SEC5
  • Q23
1
Reply
Help
Why wouldn't A be the right answer?
claire_crites on June 6, 2020
  • September 2009 LSAT
  • SEC5
  • Q21
2
Replies
Question 21
Can someone explain this problem to me please?
Samir-Ghani on June 6, 2020
  • September 2009 LSAT
  • SEC5
  • Q21
2
Replies
(A) and (D)
Hi LSAT Max, I was pretty confident answer choice (D) was correct :( After looking back, ...
Julie-V on May 26, 2020
  • September 2009 LSAT
  • SEC5
  • Q12
2
Replies
B+ C
Why are B and C incorrect?
ariella on May 12, 2020
  • September 2009 LSAT
  • SEC5
  • Q9
3
Replies
Answer C Wording
If C was reversed in that it said a whole to parts situation, where the mistake would be in assum...
layah123 on May 1, 2020
  • September 2009 LSAT
  • SEC5
  • Q5
1
Reply
How is volcanic activity not a natural cause?
Reiterating Maria's question: How is volcanic activity not a natural cause?
natnabila on April 23, 2020
  • September 2009 LSAT
  • SEC5
  • Q23
2
Replies
Explain C & A
Could you please explain how C & A are incorrect answers ? thanks a lot
Maria-Marin on April 17, 2020
  • September 2009 LSAT
  • SEC5
  • Q10
1
Reply
Confused
Please Explain
KDA86 on April 7, 2020
  • September 2009 LSAT
  • SEC5
  • Q17
2
Replies
Stuck between C and D
I understand why D is the correct answer choice but why is C incorrect?
armandovidana097 on February 11, 2020
  • September 2009 LSAT
  • SEC5
  • Q6
3
Replies